LSAT and Law School Admissions Forum

Get expert LSAT preparation and law school admissions advice from PowerScore Test Preparation.

 Administrator
PowerScore Staff
  • PowerScore Staff
  • Posts: 8916
  • Joined: Feb 02, 2011
|
#40577
Complete Question Explanation
(The complete setup for this game can be found here: lsat/viewtopic.php?t=5718)

The correct answer choice is (B)

The question stem stipulates that W and S must be shown on the same screen. While this information eliminates screen 3 from consideration for this pair (because only one movie is shown on screen 3), instead take a moment to focus on the grouping element in this question.

This is an Advanced Linear game, but all Advanced Linear games contain Grouping (from the mixing of the different variables sets), and the grouping element comes into play with the three screens. Thus, you must always carefully track information that relates to these three groups. In this question, W and S are shown on one screen. That leaves H, M, and R to be shown on the other two screens. But, from the fourth rule, H and M cannot be shown on the same screen, and thus they must be separated. With this in mind, consider just the grouping elements in play in this question (linearity is not a consideration in the groups below):
PT70 -Game_#3_#15_diagram 1.png
This leads to a Hurdle the Uncertainty inference that R is always shown on a screen with another movie (again just the grouping elements are shown in the diagram below, and linearity is not a consideration):
PT70 -Game_#3_#15_diagram 2.png
Thus, although we can’t be sure which screens H and M are shown on, we can determine that R must be shown on a screen that shows two movies.

Of course, further inferences can be drawn from the information above. First, the screen that shows a single movie must be screen 3. The screens that show two movies must be screens 1 and 2. And, because the third rule states that R cannot be shown on screen 2, we can infer that the group that contains R must be shown on screen 1, and the W and S group must be shown on screen 2:
PT70 -Game_#3_#15_diagram 3.png
Answer choice (A): This answer choice cannot occur because H must be shown on screen 1 or screen 3.

Answer choice (B): This is the correct answer choice. When this occurs, M is shown on screen 1, R is also shown on screen 1 (at 7 P.M.), and H is shown on screen 3 at 8 P.M.

Answer choice (C): This answer choice cannot occur because R must be shown on screen 1.

Answer choice (D): This answer choice cannot occur because S must be shown on screen 2 at 9 P.M.

Answer choice (E): This answer choice cannot occur because W must be shown on screen 2 at 7 P.M.
 angelsfan0055
  • Posts: 39
  • Joined: Feb 26, 2021
|
#85721
Would you be able to further explain the inference that R can't go on Screen 3? Is that for this question only or for the whole game?
 Adam Tyson
PowerScore Staff
  • PowerScore Staff
  • Posts: 5153
  • Joined: Apr 14, 2011
|
#85772
That's an inference that's local to this question, angelsfan0055. If W and S are on the same screen, then they must be on either screen 1 or screen 2. Since H and M cannot be on the same screen, one of them must be paired with R (and that would now have to be on screen 1), with the other one going to screen 3. Put another way, if R was on screen 3, then H and M would have to be on the same screen as each other, violating the last rule.

Get the most out of your LSAT Prep Plus subscription.

Analyze and track your performance with our Testing and Analytics Package.